A different way of asking questions

Algebra Level 3

If x , y x,y and z z are real numbers such that their sum is 1, find the maximum value of x y + x z + y z xy + xz + yz .


The answer is 0.333.

This section requires Javascript.
You are seeing this because something didn't load right. We suggest you, (a) try refreshing the page, (b) enabling javascript if it is disabled on your browser and, finally, (c) loading the non-javascript version of this page . We're sorry about the hassle.

6 solutions

L e t x = 1 / 3 + a , y = 1 / 3 + b , a n d z = 1 / 3 + c . x + y + z = 1 / 3 + a + 1 / 3 + b + 1 / 3 + c = 1 + a + b + c . B u t a s x + y + z = 1 , w e d e d u c e t h a t a + b + c = 0. ( a + b + c ) 2 = a 2 + b 2 + c 2 + 2 ( a b + a c + b c ) = 0 2 ( a b + a c + b c ) = ( a 2 + b 2 + c 2 ) a b + a c + b c = ( a 2 + b 2 + c 2 ) / 2 = d , w h e r e d 0 S o x y + x z + y z = ( 1 / 3 + a ) ( 1 / 3 + b ) + ( 1 / 3 + a ) ( 1 / 3 + c ) + ( 1 / 3 + b ) ( 1 / 3 + c ) = 1 / 9 + a / 3 + b / 3 + a b + 1 / 9 + a / 3 + c / 3 + a c + 1 / 9 + b / 3 + c / 3 + b c = 1 / 3 + ( 2 / 3 ) ( a + b + c ) + a b + a c + b c A s a + b + c = 0 a n d a b + a c + b c = d , w e g e t , S = x y + x z + y z = 1 / 3 d 1 / 3 = 1 / 3 + ( 2 / 3 ) ( a + b + c ) + a b + a c + b c A s a + b + c = 0 a n d a b + a c + b c = d , w e g e t , S = x y + x z + y z = 1 / 3 d 1 / 3 Let\quad x=1/3+a,y=1/3+b,and\quad z=1/3+c.\\ x+y+z=1/3+a+1/3+b+1/3+c=1+a+b+c.\\ But\quad as\quad x+y+z=1,we\quad deduce\quad that\quad a+b+c=0.\\ \therefore { (a+b+c) }^{ 2 }={ a }^{ 2 }+{ b }^{ 2 }+{ c }^{ 2 }+2(ab+ac+bc)=0\\ \quad \quad 2(ab+ac+bc)=-({ a }^{ 2 }+{ b }^{ 2 }+{ c }^{ 2 })\\ \therefore \quad ab+ac+bc=-({ a }^{ 2 }+{ b }^{ 2 }+{ c }^{ 2 })/2=-d,where\quad d\ge 0\\ So\quad xy+xz+yz\\ =(1/3+a)(1/3+b)+(1/3+a)(1/3+c)+(1/3+b)(1/3+c)\\ =1/9+a/3+b/3+ab+1/9+a/3+c/3+ac+1/9+b/3+c/3+bc\\ =1/3+(2/3)(a+b+c)+ab+ac+bc\\ As\quad a+b+c=0\quad and\quad ab+ac+bc=-d,we\quad get,\\ S=xy+xz+yz=1/3-d1/3=1/3+(2/3)(a+b+c)+ab+ac+bc\\ As\quad a+b+c=0\quad and\quad ab+ac+bc=-d,we\quad get,\\ S=xy+xz+yz=1/3-d\le 1/3

Ugghhh I was entering 0.33 \color{#3D99F6}{0.33}

Pranjal Jain - 6 years, 6 months ago

Log in to reply

I wrote a note

Abdulrahman El Shafei - 6 years, 6 months ago

Log in to reply

where are you?

U Z - 6 years, 4 months ago

I have updated the answer to 0.333

In future, if you spot any errors with a problem, you can “report” it by selecting the “dot dot dot” menu in the lower right corner. You will get a more timely response that way.

Calvin Lin Staff - 6 years, 5 months ago

Log in to reply

Thanks! I thought its my mistake not to notice the note written.

Pranjal Jain - 6 years, 5 months ago
Aritra Jana
Nov 29, 2014

First of all, this was a nice question. I have taken an unconventional way to solve this problem. enjoy :D


We Have: x , y , z R x,y,z\in\mathbb{R} and x + y + z = 1 x+y+z=1

We need to calculate the maximum value of S = x y + y z + z x S=xy+yz+zx

LET: x , y , z x,y,z be the three real roots of a monic cubic polynomial P ( a ) P(a)

By Vieta's Relations, we have:

P ( a ) = a 3 a 2 + S a + t \large{P(a)=a^{3}-a^{2}+Sa+t} where t t is a constant.

we also have: P ( a ) = 3 a 2 2 a + S \large{P'(a)=3a^{2}-2a+S}

Now for a cubic polynomial P ( a ) P(a) to have 3 real roots it should change it's direction(i.e. the sign of it’s slope \text{sign of it's slope} ) 3 times.

more importantly, thus, the curve of P ( a ) P(a) should be parallel to the X-axis 2 times \textbf{parallel to the X-axis 2 times} ...

IMPLYING THAT: P ( a ) P'(a) should be equal to 0 0 twice!

So, it is clear that P ( a ) P'(a) should have two real roots.

for P ( a ) = 3 a 2 2 a + s \therefore \text{for } P'(a)=3a^{2}-2a+s

Δ = 2 2 4.3. S 0 \large{\Delta=2^{2}-4.3.S≥0}

S 1 3 = 0.3 \implies \large{S≤\dfrac{1}{3}=\boxed{0.3}}


What do you think @abdulrahman khaled .. :D

awesome solution dude.. :)

Yash Choudhary - 6 years, 5 months ago

Creative way to answer the question

Abdulrahman El Shafei - 6 years, 6 months ago

Log in to reply

thanks :D

this was approachable in a number of ways

Aritra Jana - 6 years, 6 months ago

Log in to reply

Let x = 1/3 + a, y = 1/3 + b, and z = 1/3 + c.

therefore x + y + z = 1/3 + a + 1/3 + b + 1/3 + c = 1 + a + b + c.

But as x + y + z = 1, we deduce that a + b + c = 0.

therefore (a + b + c)2 = a2 + b2 + c2 + 2(ab + ac + bc) = 0 2(ab + ac + bc) = -(a2 + b2 + c2) therefore ab + ac + bc = -(a2 + b2 + c2)/2 = -d, where d greater than or equal 0

So xy + xz + yz = (1/3 + a)(1/3 + b) + (1/3 + a)(1/3 + c) + (1/3 + b)(1/3 + c) = 1/9 + a/3 + b/3 + ab + 1/9 + a/3 + c/3 + ac + 1/9 + b/3 + c/3 + bc = 1/3 + (2/3)(a + b + c) + ab + ac + bc As a + b + c = 0 and ab + ac + bc = -d, we get, S = xy + xz + yz = 1/3 - d less than or equal 1/3 Q.E.D..

Parth Lohomi - 6 years, 6 months ago

Log in to reply

@Parth Lohomi absolutely correct :D

Aritra Jana - 6 years, 6 months ago

Log in to reply

@Aritra Jana Thnx XD.................

Parth Lohomi - 6 years, 6 months ago

Very good approach

U Z - 6 years, 5 months ago
Kristian Vasilev
Dec 4, 2014

We\quad know\quad that\quad for\quad real\quad numbers\quad x,y\quad and\quad z\quad { (x-y) }^{ 2 }+{ (x-z) }^{ 2 }+{ (y-z) }^{ 2 }\ge 0.\\ Therefore\quad { x }^{ 2 }+{ y }^{ 2 }+{ z }^{ 2 }\ge xy+yz+zx.So\quad { x }^{ 2 }+{ y }^{ 2 }+{ z }^{ 2 }+2xy+2yz+2zx\ge 3xy+3yz+3zx.Therefore\quad \\ { (x+y+z) }^{ 2 }\ge 3S.\gg \quad \frac { 1 }{ 3 } \ge S.So\quad the\quad final\quad answer\quad is\quad 0.3\quad

nice solution

Omar El Mokhtar - 6 years, 6 months ago
Kartik Sharma
Nov 30, 2014

I really think that this is overrated.

( x + y + z ) 2 = x 2 + y 2 + z 2 + 2 ( x y + y z + z x ) {(x + y + z)}^{2} = {x}^{2} + {y}^{2} + {z}^{2} + 2(xy + yz + zx)

( x + y + z ) 2 ( x 2 + y 2 + z 2 ) 2 = x y + y z + z x \frac{{(x + y + z)}^{2} - ({x}^{2} + {y}^{2} + {z}^{2})}{2} = xy + yz + zx

Now we have to just find the minimum value of the LHS.

LHS = 1 ( x 2 + y 2 + z 2 ) 2 \frac{1 - ({x}^{2} + {y}^{2} + {z}^{2})}{2}

Using Cauchy Schwarz like this -

( x 2 + y 2 + z 2 ) ( 1 2 + 1 2 + 1 2 ) ( x + y + z ) 2 ({x}^{2} + {y}^{2} + {z}^{2})({1}^{2} + {1}^{2} + {1}^{2}) \geq {(x+y+z)}^{2}

( x 2 + y 2 + z 2 ) 1 3 ({x}^{2} + {y}^{2} + {z}^{2}) \geq \frac{1}{3}

LHS 1 1 3 2 \le \frac{1 - \frac{1}{3}}{2}

1 3 \le \frac{1}{3}

Hence, x y + y z + z x 1 3 xy + yz + zx \le \frac{1}{3}

(x+y+z)^{2} =x^{2}+y^{2}+z{2}+2(xy +yz+zx).

let x^{2}+y^{2}+z{2} be a^{2},

a^{2}>=0

2(xy+yz+zx)=1-a^{2}

2S<=1

hence, S<=1/2

S<=0.5

Arpit Agrawal - 6 years, 6 months ago

Same approach. Also do think it's a bit overrated.

Ryan Tamburrino - 6 years, 5 months ago
Angelo Forcadela
Nov 27, 2014

x+y+z=1, pretend x=y=z=1/3, S=1/9+1/9+1/9 = 1/3 =0.333333333333333333.....

you can't pretend that x=y=z it isn't given.

Abdulrahman El Shafei - 6 years, 6 months ago

Log in to reply

عبد الرحمان ،جزاك الله خيرا احتاج الى كتاب في الاولمبياد.

Omar El Mokhtar - 6 years, 6 months ago

Log in to reply

@Omar El Mokhtar Please express your views only in English.

Anuj Shikarkhane - 6 years, 6 months ago

Log in to reply

@Anuj Shikarkhane i need some books for olympiad,can you help me?

Omar El Mokhtar - 6 years, 6 months ago

Log in to reply

@Omar El Mokhtar Can you please specify for which olympiad you want books?

Anuj Shikarkhane - 6 years, 6 months ago

Log in to reply

@Anuj Shikarkhane thanks,i want math olympiade math books please

Omar El Mokhtar - 6 years, 6 months ago
Shubham Kumar
Aug 11, 2015

as ( x+y+z)^2 = x^2 + y^2 + z^2 + 2S so x^2 + y^2 + z^2 =1 - 2S Now (x-y)^2 + (y-z)^2 +(z-x)^2 = 2( x^2 + y^2 + z^2) - 2S i.e. 2(1-2S) - 2S >=0 or 1 - 3S >= 0 Hence S <= 1/3

0 pending reports

×

Problem Loading...

Note Loading...

Set Loading...